Fourier Reihe?


06.06.2023, 21:32

Verbessert


06.06.2023, 22:08

Angabe


07.06.2023, 16:20

Zusatz

1 Antwort

Vom Fragesteller als hilfreich ausgezeichnet

Mit etwas freier Interpretation und hoffentlich keinen Rechenfehlern komme ich auf das Ergebnis (beachte dass keine Kosinusterme wegen f(–x)=–f(x) berechnet werden müssen, da sie null sind):

Bild zum Beitrag

Korrektur zur letzten Zeile:

b_n = 1/pi * ( (–1)^n * 2/n + 1/n + 1/n + (–1)^n * 2/n )

b_n = ((–1)^n * 4 + 2) / (pi * n)

Überall müsste 1/pi statt 1/(2 * pi) stehen.

Edit: Die Rechnung ist fehlerbehaftet.

Woher ich das weiß:Hobby – Mathematik (u. Physik)
 - (Funktion, Gleichungen, Mathematiker)
Nilson2023 
Fragesteller
 06.06.2023, 15:57

Danke für deine Antwort!

Ich habe im Bsp. aber f(-x) = - f(x) und auch T = 2pi vorgeben.

Der Vorfaktor ist doch 2/T oder nicht? Somit komme ich eben auf die 1/pi als Vorfaktor.

Und nochmal kurz zu den Grenzen das passt aber schon so wie die gesetzt sind oder sprich von 0 bis pi/2 und von pi/2 bis pi?

Danke nochmal

0
TBDRM  06.06.2023, 18:41
@Nilson2023

Die Grenzen passen, ja. Die gehen eben über die Periode. Die ist aber pi und nicht 2 * pi (die Vorgabe ist Quatsch, siehe auch Skizze).

0
Nilson2023 
Fragesteller
 06.06.2023, 21:06
@TBDRM

Sicher?

Mir wurde nämlich gesagt, dass die Skizze (die übrigens von mir und nicht aus der Angabe ist) falsch ist und auch beim a0 falsch ist.

Wenn ichs rechne, dann komme ich jedoch immer aufs selbe Ergebnis, aber ja vielleicht passt es ja dann doch so wie du sagst.

0
Nilson2023 
Fragesteller
 06.06.2023, 21:34
@TBDRM

So ich hab das Bsp. Jetzt nochmal ergänzt und L definiert, L ergibt sich ja aus T und ist in diesem Fall dann ja pi.

Wenn ich L dann in meiner Formel so anwende ist das korrekt oder?

0
TBDRM  06.06.2023, 21:47
@Nilson2023

Kannst du mal ein Foto der originalen Aufgabe hochladen. Denn entweder hast du irgendwas falsch übernommen an Informationen oder ich vertue mich gerade voll.

0
Nilson2023 
Fragesteller
 06.06.2023, 22:05
@TBDRM

Okay ja passt ich hab die Angabe hochgeladen sollte sichtbar sein

1
TBDRM  06.06.2023, 22:59
@Nilson2023

Also für mich macht die Aufgabe keinen Sinn. Wie soll Punktsymmetrie zum Urspung herrschen, wenn f für Stellen kleiner null gar nicht mal definiert ist?

Das einzige sinnvolle ist, die Funktion gedanklich bezüglich diesen Eigenschaften zu erweitern.

Ich versuche mal eben eine eigene Lösung hochzuladen.

0
Nilson2023 
Fragesteller
 07.06.2023, 10:36
@TBDRM

Danke vielmals für deine Mühe!

Ich schaue mir das ganze dann an wenn ich zuhause bin :)

1
TBDRM  07.06.2023, 12:42
@Nilson2023

Habe allerindings einen Fehler in der letztes Zeile gemacht. Werde unter der Antwort ne Korrektur schreiben.

0
Nilson2023 
Fragesteller
 07.06.2023, 14:53
@TBDRM

Alles klar, heißt aber in deiner Schreibweise für bn nimmst du dann gleich den periodischen Lauf mit, (bessere Näherung) also weil du ja bei - pi anfängst.

Wäre es hier falsch die Grenzen von 0 bis pi/2 und von pi/2 zu pi zu setzen, sowie es vorgeben ist? Kommt ja wahrscheinlich aufs gleiche oder?

0
Nilson2023 
Fragesteller
 07.06.2023, 16:17
@TBDRM

Also ich hab das jetzt nochmal durchgemacht, bis zur Integration ist alles gleich aber dann..

Zum Schluss ja einfach normal einsetzten und vereinfachen - fertig.?

Dann kommt halt das dabei raus 😬

Ich hab meine Rechnung oben ergänzt

0
TBDRM  07.06.2023, 17:08
@Nilson2023

Deine zweite Zeile ist komplett falsch. Bei mir habe ich aber auch ähnlich Fehler.

1. Integral

–2 * cos(nx) / n |{–pi/2; –pi}

= –2/n * ( cos(–pi/2 * n) – cos(–pi * n) )

= –2/n * ( cos(pi/2 * n) – cos(pi * n) )

= –2/n * ( g(n) – (–1)^n )

= –2/n * g(n) + (–1)^n

mit g(n) = 0 wenn n ungerade und g(n) = (–1)^(n/2) wenn n gerade. Ich weiß nicht, wie man das als geschlossenen Ausdruck darstellen kann. Vielleicht kann man ja zwei Summen bilden.

0
TBDRM  07.06.2023, 17:13
@Nilson2023

Meine Rechnung ist auch falsch. Ich weiß gerade leider nicht weiter.

0
Nilson2023 
Fragesteller
 07.06.2023, 17:21
@TBDRM

Schade, echt komisches Beispiel... Und das ist erst das erste von vier in diesem Übungsblatt 🥴

Danke dir jedenfalls für die super Hilfe, hat mir echt schon gut geholfen :)

1
Nilson2023 
Fragesteller
 07.06.2023, 23:42
@Nilson2023

Info: a0 kann man direkt zu Null setzten, da ungerade, dann sollte das Beispiel passen

0